Тёмный

FUN Ecuadorian Math Olympiad Number Theory Problem 

Michael Penn
Подписаться 305 тыс.
Просмотров 43 тыс.
50% 1

Normally, there'd be some WILD stuff here, and maybe there still is, but hear me out first...
This is a new and occasional format that we are trying out for the first time and absolutely want your feedback so, please, actually watch the entire video from start to finish and sound off in the comments about what you think of it.
If you don't do the above I cannot guarantee that Chalkboard, her best pal Eraser, and even the dastardly Chalk won't break down in a fit of uncontrollable wail-weeping whereby wispy winds wind willingly with well-wishing wizards wearing wetsuits, watches, wigs - whole wardrobe! Watch the dang video till the end and comment pls ok thx. It'd mean a lot to me, PLUS, did you see that sentence with all those words starting with "w". That's worth something right? RIGHT? Okay anyway, watch video, comment your thoughts on the new format.
-Stephanie
MP Editor
🌟Support the channel🌟
Patreon: / michaelpennmath
Merch: teespring.com/...
My amazon shop: www.amazon.com...
🟢 Discord: / discord
🌟my other channels🌟
mathmajor: / @mathmajor
pennpav podcast: / @thepennpavpodcast7878
🌟My Links🌟
Personal Website: www.michael-pen...
Instagram: / melp2718
Twitter: / michaelpennmath
Randolph College Math: www.randolphcol...
Research Gate profile: www.researchga...
Google Scholar profile: scholar.google...
🌟How I make Thumbnails🌟
Canva: partner.canva....
Color Pallet: coolors.co/?re...
🌟Suggest a problem🌟
forms.gle/ea7P...
#prime #primes #cubes #cubicequation #numbertheory

Опубликовано:

 

26 сен 2024

Поделиться:

Ссылка:

Скачать:

Готовим ссылку...

Добавить в:

Мой плейлист
Посмотреть позже
Комментарии : 139   
@tomholroyd7519
@tomholroyd7519 Год назад
He has room for backflips now
@bini420
@bini420 Год назад
the comment section could pressure him into doing backflips at the beginning of every video
@xl000
@xl000 Год назад
he is a rock climber, not a gymnast. It explains the size of his forearms and biceps
@brabhamfreaman166
@brabhamfreaman166 8 месяцев назад
Didn’t seem a hindrance before
@lexyeevee
@lexyeevee Год назад
i love this very formal number theoretic proof of a lemma that i would've summarized as "well it's gotta be somewhere"
@hybmnzz2658
@hybmnzz2658 Год назад
Oh man you are giving me flashbacks to my number theory prof ranting about how mathematicians in the 1600s-1800s would assume p|ab implies p|a or p|b by assuming unique factorization into primes. Apparently only Gauss cared enough to note it's circular reasoning.
@JM-us3fr
@JM-us3fr Год назад
@@hybmnzz2658 Euclid also cared, since he proved unique factorization
@JM-us3fr
@JM-us3fr Год назад
I think the “well it’s gotta be somewhere” proof assumes unique factorization. Nonetheless, that proof still becomes important in integral domains, when we prove that in UFDs all irreducibles are prime.
@tracyh5751
@tracyh5751 Год назад
@@hybmnzz2658 Eh, that depends heavily on how you define a number to be a prime. For commutative rings for example, p|ab -> p|a or p|b is the definition for a prime in any Principal Ideal Doman (the integers being an example of a PID).
@davidemasi__
@davidemasi__ Год назад
I love it when you solve this kind of problems and teach such nice tricks which can be applied in similar situations and I also appreciate the idea of the two blackboards. The channel is getting better and better, I'm sure you'll get to pi-thousand subscribers soon!
@Bodyknock
@Bodyknock Год назад
Working through the bonus problem where p² + p + 1 = n³, the steps are mostly the same but with an occasional sign flip. Suppose p(p+1) = n³ - 1 = (n-1)(n² + n + 1). Then either p|(n-1) or p|(n² + n + 1) Case 1 - Suppose p|(n-1), i.e. n-1 = px for some natural x. Then p ≤ n-1 which implies p+1 ≤ n But also p(p+1) = px(n² + n + 1) which implies p+1 = xn² + xn + x ≥ n² + n + 1 > n (since x and n are Natural numbers) So n ≥ p+1 > n, which is a contradiction. Therefore p does not divide (n-1) Case 2 - Suppose p | n² + n + 1, i.e. n² + n + 1 = px for some natural x Then p(p+1) = px(n-1), so p = xn - x - 1 Plugging p in, we get n² + n + 1 = nx² - x² - x Solving for x, we find that the discriminant f(x) = x⁴ - 6x² - 4x - 3 must be a perfect square. (Similar to the video except we have a -4x term instead of a +4x term.) Similar to the video, set g(x) = (x² - 4)² and h(x) = (x² - 3)² (so instead of 3 and 2 we're using 4 and 3 as the differences for the consecutive squares). f(x) - g(x) = 2x² - 4x - 19. Note that if you solve for x this is >0 if x>4 or x4 part. h(x) - f(x) = 4x + 12, which is always >0 for all Natural x. Therefore if x>4 we have h(x) > f(x) > g(x), which puts f(x) between two consecutive squares. So there can be no solutions where x>4 (just like the video, only with slightly different consecutive squares.) So the only possible solutions are x = 1, 2, 3, or 4and only if f(x) is itself a perfect square. Recall above f(x) = x⁴ - 6x² - 4x - 3 for reference f(1) = 1-6-4-3 = -12, not a perfect square f(2) = 16 - 24 - 8 - 3 = -19, not a perfect square f(3) = 81 - 54 - 12 - 3 = 12, also not a perfect square f(4) = 141, not a perfect square Therefore f(x) is never a perfect square for any natural number x, and thus there are no solutions.
@someperson188
@someperson188 Год назад
It should be mentioned that n 1 (which is easily checked) in order to conclude (in Case 1) that p | n -1 implies n -1 = px, where x is a natural number. f(x) - g(x) = 2x² - 4x - 19 = 2x(x - 2) - 19 > 0, for x >= 5. So, you must also check that f(4) = 141 isn't a perfect square. I think you meant "solving for n" not "solving for x".
@Bodyknock
@Bodyknock Год назад
@@someperson188 Which is not a square.
@georgelaing2578
@georgelaing2578 Год назад
Your arithmetic is perfect, except that f(x) - g(x) should end in -19. The expression is positive when x is strictly greater than 4, so the logic still holds.
@Bodyknock
@Bodyknock Год назад
@@georgelaing2578 Thanks, I corrected that and also added the f(4) case that needs to be explicitly checked.
@leif1075
@leif1075 Год назад
Whyvdkesbt hevjustbuse this that you can rewrite as p(p-1)= n^3-1 and use that tpnsolve instead of a lemma I don't think most ppl have heard of or would deduce anyway if theyvhavent..cantnyoubthebsplve by saying either p equals n minus 1 and p minus 1 =(n^2 + n +1) or vice versa and you are done?
@andrewlitfin1977
@andrewlitfin1977 Год назад
I actually really like the new format! Feels a lot smoother
@pingpongfulldh2308
@pingpongfulldh2308 Год назад
I never understood why this channel only has 251k, this is a gem!
@funatish
@funatish Год назад
So Stephanie is the name of our unsung heroine of the description short stories. Shoutout to her!
@MichaelPennMath
@MichaelPennMath Год назад
I'm also the editor lol :) -Stephanie MP Editor
@zuzaaa1998
@zuzaaa1998 Год назад
I think that this new format has some potential but there are a few problems. The lighting is a little bit off and some parts of the board are more illuminated than others. Other thing is that you have always had previous steps summarised somewhere in the corner and now despite the fact that you have more space some previous facts were erased and it was harder to recall what was already done. Like the equation for n in terms of x and the problem statement itself that was only on one of the boards
@maldi_tof2910
@maldi_tof2910 Год назад
Love how you are choosing problems that tie into your MathMajor channel and the recent proof writing series. Do continue this trend of having your problems tie into your lectures. Love it.
@vaxjoaberg
@vaxjoaberg Год назад
I like the new format.
@mcwulf25
@mcwulf25 Год назад
Thorough as ever. The first p|ab bit can be easily proven with prime factorisation.
@confidential9411
@confidential9411 Год назад
It first appeared in St. Petersburg olympiad in 90's, then appeared in Balkan Olympiad
@mMaximus56789
@mMaximus56789 Год назад
I would love a series from you doing more 'modern' multivariable calculus as to arrive to the generalized stokes theorem with differential forms
@DavidSavinainen
@DavidSavinainen Год назад
The studio looks awesome! I also like this problem a lot; Using trial and error on p, 19 is quite large (even for a prime, there's seven smaller ones) so if one tested seven or fewer of them, one would like to conclude that there is no solution, however in such a situation, one would be wrong.
@ttrss
@ttrss Год назад
production quality is insane
@karimshariff7379
@karimshariff7379 Год назад
Really nice problem and solution. Two blackboards look optimal.
@gazzamgazzam4371
@gazzamgazzam4371 Год назад
Hello everyone, p^2-p+1=n^3...(*), as you said there exists a natural x>0 s.t. px=n^2+n+1 ....(1) p-1=×(n-1) .......(2) we have from (1): px=(n-1)(n+2)+3 multiplying both sides by x, one obtains: px^2=x(n-1)(n+2)+3x . Using (2), one can write: px^2=(p-1)(n+2)+3x. This equation is equivalent to , p(x^2-n-2)=3x-n-2 (it is easy to prove the positivity of both sides). For x>3, we have : 3x^2-n-2>3x-n-2 and since p is a prime (greater than 1) the last equation does not have a solution. Now we need to check the last equation for the cases: x=1, x=2, and x=3. By easy calculation, the equation (*) has a solution for the case x=3 while there are not solutions for the two other cases. For x=3, one can easily determine p and n which they are : p=19 and n=7. The answer to the question is : there is only one prime which is 19. Thank you for this good problem.
@boluo7157
@boluo7157 Год назад
amazing quality and production, keep up the good work!
@EyadAmmari
@EyadAmmari 8 месяцев назад
Beautiful!
@0xTJ
@0xTJ Год назад
I love the look of the space!
@runzeli1392
@runzeli1392 Год назад
I like your video and the format! Please keep going! Thank you!
@robshaw2639
@robshaw2639 Год назад
I don't find any primes less than 10,000 for which p^2 + p + 1 works
@Bodyknock
@Bodyknock Год назад
That checks, I solved it using the same method as the video and the corresponding f(x) is never a perfect square for any natural x in the bonus problem, meaning no solutions.
@phee4174
@phee4174 Год назад
15:31 this is a bit of minor nitpicking , but -3 is in fact a perfect square, if one works over the Eisenstein integers , specifically it's (1+2ω)^2
@alvarezjulio3800
@alvarezjulio3800 Год назад
Beautiful problem and excelent problem solving!
@Dr.1.
@Dr.1. Год назад
Wow amazing video... Love the new format
@motoroladefy2740
@motoroladefy2740 Год назад
What a setup man!
@LaszloBattha
@LaszloBattha Год назад
The following solution maybe of interest since it only uses basic facts about divisibility, and the simple observation: if (4i+r)(4k+s)=4l+t, where 0=2 and thus x>0. But then (n-1)(n-2)>= p(n-2)=p(px-1)>= p(p-1)=(n-1)m>m and thus 0>-n^2+3n-2+m=4n-1>=11, a contradiction. Hence m=px for some x>0 and thus p(p-1)=(n-1)px, i.e., p-1=(n-1)x. Hence m=((n-1)x+1)x. From 0==n(n+1)=(n-1)x^2+x-1==(n-1)x+x-1=nx-1 (mod 2) we get n=2k+1 for some k>0 & x=2y+1 for some y>=0, and thus m-1=n^2+n=4k^2+6k+2=2(2k+1)(k+1) & m-1=((n-1)x+1)x-1=2k(2y+1)^2+2y=8ky^2+(8k+2)y+2k, i.e., 2k(k+1)+1=y(4ky+4k+1). From 2 | k(k+1) we get y=4z+1 for some z>=0 and thus 2k^2+2k+1=2k(k+1)+1=(4z+1)(4k(4z+1)+4k+1)=4k(4z+1)^2+ 16kz+4z+4k+1, i.e., k^2=2k(4z+1)^2+8kz+2z+k. Hence 2z=kl for some l>=0 and thus k^2=2k(2kl+1)^2+(4k+1)kl+k, i.e., k=2(2kl+1)^2+(4k+1)l+1. Assume(!) that l>=1. Then k>=2(2k+1)^2+4k+1+1>=2*3^2+4k+2, a contradiction. Hence l=0 and thus k=3 & z=0 & y=1 & x=3. This gives n=2k+1=7 & p-1=(n-1)x=6*3=18.
@roneyandrade6287
@roneyandrade6287 Год назад
Hey I'm Ecuadorian!
@romajimamulo
@romajimamulo Год назад
The colors are excellent by the way
@ManuelRacle
@ManuelRacle Год назад
I did really like this one! Thanks
@thsand5032
@thsand5032 Год назад
When getting to p | n²+n+1, I tried seeing if there was any way to use Eisenstein integers to solve the problem. You can immediately deduce that p cannot be an Eisenstein prime, because if it were it would divide either n-j or n-j², meaning n-j = p(a+bj), so pb = -1. Not happening. Therefore, p = qq* for some Eisenstein prime q. From there, I was kind of stuck (I didn't try very hard though). If anyone comes up with anything coming from that, I'd be interested to hear about it.
@FullAfterburner
@FullAfterburner Год назад
37(n+n+n)=nnn for 0 < n < 10. Sin (666) + Cos (6*6*6) = -1.618... (golden ratio). Belphegor's prime is the palindromic prime number 1000000000000066600000000000001 (10^30 + 666 × 10^14 + 1).
@manucitomx
@manucitomx Год назад
Thank you, professor and congratulations on the new set-up. Now that there is room, are the backflips making a comeback?
@krisbrandenberger544
@krisbrandenberger544 Год назад
Hey, Michael! The constant term of h(x)-f(x) should be positive seven, not negative seven.
@jellyfrancis
@jellyfrancis Год назад
Nice studio 👌💥❤️
@nathanaschmann7522
@nathanaschmann7522 Год назад
Cool math coming from my home country, cool.
@hazalouldi7130
@hazalouldi7130 Год назад
you are right
@howareyou4400
@howareyou4400 Год назад
I wonder why we spent time prove this lemma, while it's obvious from the "Fundamental theorem of arithmetic" by the factorization of a and b which gives the factorization of ab.
@noelwass4738
@noelwass4738 8 месяцев назад
I suppose the lemma could be the considered as the basis for proving the Fundamental Theorem of Arithmetic.
@Happy_Abe
@Happy_Abe Год назад
That “W” alliteration though
@spazmoidectomorf6209
@spazmoidectomorf6209 Год назад
If you wondering like I was, why can't p divide both n-1 and n²+n+1 its because this implies RHS has factor of p², but LHS has factor of only p which wouldn't work
@serhiislobodianiuk776
@serhiislobodianiuk776 Год назад
Nice! Did you know that there is quite similar problem from Romanian Masters 2023? Solve x^3+y^3=p(xy+p) where p is prime, x, y are positive integers
@kingfrozen4257
@kingfrozen4257 4 месяца назад
the lemma proof is circular!
@bilalabbad7954
@bilalabbad7954 Год назад
I like your videos I have learned a lot of formulas from your courses Thank you so much
@beautifulworld6163
@beautifulworld6163 Год назад
Really good❤
@torbjornolsson9087
@torbjornolsson9087 Год назад
Nice bound on squares. I came to equation through simillar reasoning that (a^2-3)^2+4(a-3)=(a+x)^2 from wich I suspektes only a=3 leading to x=3 is possible solution. Otherwise one could investigate integers u^2+v^2=z^2 maybe?
@brabhamfreaman166
@brabhamfreaman166 8 месяцев назад
6:15 If we’re working with natural numbers, you should’ve dispatched (easily, I admit, but necessary nonetheless) with the case n=1 when n-1=0 and p|0 🥴
@rosiefay7283
@rosiefay7283 Год назад
0:33 But you needn't prove that. That is the definition of "prime".
@koenth2359
@koenth2359 Год назад
For the homework p^2+p+1 = n^3 I get: p(p+1)=(n-1)(n^2+n+1) case 1) p|n-1 => n+1 n=1/2 (x^2-1+/- sqrtD) with D=x^4-6x^2-4x-3 Whenever x>=5 we have (x^2-4)^2 < x^4-6x^2-4x-3 < (x^2-3)^2, so try x=1, D=-12 x=2, D=-19 x=3, D=12 x=4, D=256-96-16-3=141 so D is never a perfect square and there are no solutions
@jeremydavie4484
@jeremydavie4484 Год назад
I love your videos! What is the best way to tackle writing proofs without getting any hints? Does it come with time? Where do most math people get these crazy intuitions to use certain lemmas? Thank you.
@bosorot
@bosorot Год назад
2 boards are good. But do you know what is better? . Make a 3rd board on the ceiling. I doubt that Michael can climb !!!
@MichaelPennMath
@MichaelPennMath Год назад
What if I told you he's an avid rock climber? lol -Stephanie MP Editor
@catalogapp5776
@catalogapp5776 Год назад
​@@MichaelPennMath I think that it was a pretty funny joke based on this fact
@armandohernandez5329
@armandohernandez5329 Год назад
Hi everyone. There is a writing mistake: h(x)-f(x)=2x^2-4x+7 for x>=4
@LaszloBattha
@LaszloBattha Год назад
Still not correct, but now i have it: if (4i+r)(4k+s)=4l+1, where 0
@gregevgeni1864
@gregevgeni1864 Год назад
very nice ❤
@LaszloBattha
@LaszloBattha Год назад
Sorry, the correct form is: if (4i+r)(4k+s)=4l+1, where 0
@DavidVonR
@DavidVonR Год назад
I came up with a brute force solution by mentally checking all primes less than 100. The only values I could find were p = 19, n = 7. I tried to solve this one algebraically, got nowhere.
@confidential9411
@confidential9411 Год назад
The lemma is evident fact
@aymantimjicht173
@aymantimjicht173 Месяц назад
A faster way to prove p1|ab ⇒ p1|a or p1|b: Let a = ∏p^α1 and b = ∏p^α2, where p represents prime factors. If p1|ab, then the exponent of p1 in the prime factorization of ab is positive. This means α1 + α2 > 0 for p1. Therefore, at least one of α1 or α2 must be positive for p1. So, p1|a or p1|b. Proving Prime Factor Divisibility Here's the corrected text with explanations: A faster way to prove p1|ab ⇒ p1|a or p1|b: Let a = ∏p^α1 and b = ∏p^α2, where p represents prime factors. If p1|ab, then the exponent of p1 in the prime factorization of ab is positive. This means α1 + α2 > 0 for p1. Therefore, at least one of α1 or α2 must be positive for p1. So, p1|a or p1|b. I Would you explain any part of this proof in more detail Certainly, I'd be happy to explain this proof in more detail. Let's break it down step by step: Prime factorization: We start by expressing a and b in terms of their prime factorizations: a = ∏p^α1 and b = ∏p^α2 This means a and b are the product of prime factors (p) raised to certain powers (α1 and α2 respectively). The meaning of p1|ab: When we say p1|ab, we mean p1 divides ab. In terms of prime factorization, this means p1 appears as a factor in the prime factorization of ab. Exponents in multiplication: When we multiply a and b, the exponents of like prime factors add. So in ab, the exponent of p1 would be α1 + α2. Positive exponent: For p1 to divide ab, its exponent in the prime factorization of ab must be positive. This means α1 + α2 > 0 for p1. Logical conclusion: If α1 + α2 > 0, then at least one of α1 or α2 must be positive (or both could be positive). It's impossible for both to be zero or negative and still sum to a positive number. Final deduction: If α1 > 0, then p1 appears in the prime factorization of a, so p1|a. If α2 > 0, then p1 appears in the prime factorization of b, so p1|b. Therefore, we can conclude that if p1|ab, then p1|a or p1|b (or possibly both). This proof uses the fundamental theorem of arithmetic (unique prime factorization) and basic properties of exponents and divisibility to establish the result quickly and elegantly. I will pulblished in an other moment. Now the answer is in a comment. I prefer this way its more safer. Mafia can nakes me made for a comment just for hate. Any way is not possible to admet all those crames. They want to introduce wrong knowledge to cover tge ways of manupilation that they use. They can create Earthquakes change meteo and sayi g that is not predictible. And they want to makes me made for My article published. Even Microsoft and Alphabet is implicated. I think Opening an investigation Is possible because we can find proofs of crimes
@euleri0
@euleri0 Год назад
spot on!!
@shrayanpramanik8985
@shrayanpramanik8985 Год назад
I think I saw this kind of format in a ug entrance exam in India called the CMI entrance. I'd suggest you to have a look at those papers! You'd like them.
@physicorum7107
@physicorum7107 Год назад
Which year's paper mate ?
@shrayanpramanik8985
@shrayanpramanik8985 Год назад
@@physicorum7107 I'm really not sure bruv. But I'm definitely sure about what the question was. It was p³-p= n⁷-n³. Where p is a prime and n is any natural number. The solution almost exactly used this concept in the video.
@mantisbog
@mantisbog Год назад
Do Steiner math.
@TheQEDRoom
@TheQEDRoom Год назад
I know that p=3 is not a solution, but shouldn't we also check that case? Since p could divide both n-1 and n^2+n+1 and that happens when p=3.
@cameronspalding9792
@cameronspalding9792 Год назад
@ 14:14 it should be (x^2-2)^2
@narayansareekuthir5330
@narayansareekuthir5330 2 месяца назад
Do by bound that will easy its a easy problem
@gp-ht7ug
@gp-ht7ug Год назад
The two consecutive squares come out of the blue. Why have you chosen those squares?
@javiergilvidal1558
@javiergilvidal1558 Год назад
Yeah, the solution is quite contrived, and far from obvious. It DOES depend a lot on you being lucky enogh to find quite a few saving ideas! I don't think it's a fair problem ....
@KC18236
@KC18236 Год назад
😮😮😮niiice problem
@warrengibson7898
@warrengibson7898 Год назад
Were students expected to solve this problem from a standing start in an exam room?
@minwithoutintroduction
@minwithoutintroduction Год назад
رائع قبل المشاهدة
@subpopulations
@subpopulations Год назад
I misread the thumbnail at first and thought it was when is sqr(p) - p + 1 = a prime which is an interesting question, but I couldn't see an obvious pattern, I got to p has to be 6m+1 as for 6m -1 it is divisible by 3 but there appear to be an infinite number of solutions, thus I thought it seemed a little too difficult for an Olympiad question, so I went back to to check the video and noticed my mistake. 2 3 3 7 7 43 13 157 67 4423 79 6163 My Solution to the actual: so first note cube(n) = 1 (mod p) thus 3 divides p-1 (Fermat's little theorem) as n is not 1 and less than p thus p = 6m + 1 then cube(n) = 1 (mod 6) so n = 6q + 1 sqr( 6m + 1) - 6m = cube(6q + 1) 36sqr(m) + 6m + 1 = cube(6q)+ 3sqr(6q) + 3(6q) +1 some manipulation produces 6sqr(m) + m = 36cube(q) +18sqr(q) + 3q thus m = 3k and p = 18k +1 18sqr(k)+k = 12cube(q) +6sqr(q) + q pk = q(12sqr(q) +6q + 1) gcd(q, 12sqr(q) +6q + 1) =1 and gcd(p, k) =1 so q divides p or 12sqr(q) +6q + 1 divides p but p prime so if a number divides p then that number = 1 or p 12sqr(q) +6q + 1 can't be 1 so q = 1 so pk = 19 thus k =1 and p = 19 and n = 7 I saw a comment about a Test Problem: sqr(p) + p + 1 = a cube similarly 3 divides p-1 thus p = 6m + 1 then cube(n) = 3 (mod 6) so n = 6q + 3 thus cube(n) = 0 (mod 9) but sqr(p) = 1 + 12m (mod 9) then sqr(p) + p = 2 (mod 9) and so sqr(p) + p + 1 = 3 (mod 9) contradiction no such p and n exist
@nickkrempel5888
@nickkrempel5888 Год назад
You went wrong at the point you said "so q divides p or 12sqr(q) +6q + 1 divides p". That doesn't follow from the gcd conditions you established.
@abhitomar320
@abhitomar320 Год назад
Great❤
@helioserioleaodecarvalho
@helioserioleaodecarvalho Год назад
h(x) - f(x) = 2x² - 4x + 7 ?
@ilyashick3178
@ilyashick3178 Год назад
Not clear to me as well.
@johns.8246
@johns.8246 Год назад
How about p^2+p+8=n^3? I already found a solution by guess and check. But are there others?
@conntoolbox
@conntoolbox Год назад
yes, p=31... and can you find the solution to p^2 - p - 8 = n^3 ?
@johns.8246
@johns.8246 Год назад
@@conntoolbox p=1801 Don't ever do that to me again.
@Macieks300
@Macieks300 Год назад
Couldn't that first lemma be proven in one step by just using the fundamental theorem of arithmetic?
@dogedev1337
@dogedev1337 Год назад
that's circular reasoning
@notfeelin6610
@notfeelin6610 Год назад
all the proofs of fta i have seen use p|ab implies p|a or p|b. i suppose it might be possible to prove it without using that though
@PhysicsNg
@PhysicsNg Год назад
I remember this math problem pretty easy for even 9th grade
@davidwright5719
@davidwright5719 Год назад
Why isn’t the proof of lemma just… write out the prime factorizations of a and b. What is it with this guy’s love of lengthy, obfuscating proofs?
@iljas275
@iljas275 Год назад
Is it possible to prove the same if P is not a prime, but any positive integer? It seems. that result should be the same. If it is possible, then it is a part of positive integer solution of the equation y^2-x^3=1 in real numbers.
@mcwulf25
@mcwulf25 Год назад
More difficult because if p is not prime then we can't say it divides a, b or both. One factor might divide a and the other factor might divide b.
@jiioannidis7215
@jiioannidis7215 Год назад
Not exactly rigorous proof, but: Lemma: all math-olympiad problems that ask you to find all numbers having a certain property have a finite set of answers :) (or a formula, but there is no formula for prime numbers [that we know of :) ]. Proof by exhaustive search of all problems in all math olympiads :) Therefore, we start enumerating prime numbers and stop when 19 is reached. The proof that only one exists is left as an exercise to some other troll :)
@georgelaing2578
@georgelaing2578 Год назад
The long shots of your new environment are quite dramatic!!!
@qing6045
@qing6045 Год назад
What is the brand of your chalk?
@lgooch
@lgooch Год назад
Do you have to prove the first lemma in the real Olympiad or can you just state it?
@reubenmckay
@reubenmckay Год назад
Michael's previous solve of the same question can be found here: ru-vid.com/video/%D0%B2%D0%B8%D0%B4%D0%B5%D0%BE-mYCVZitGsFQ.html Personally, I think his newer solution is cleaner and more elegant.
@charleyhoward4594
@charleyhoward4594 Год назад
guy's too smart for me ...
@adamnevraumont4027
@adamnevraumont4027 Год назад
Case 1 you neglected n=1 case, in which case x is 0. It doesn't work, but you didn't rule it out.
@jonathanparkes9772
@jonathanparkes9772 Год назад
Am I supposed to understand whats going on?
@alexandermorozov2248
@alexandermorozov2248 Год назад
p=19, n=7 🤪
@bini420
@bini420 Год назад
niceeeeeeeeeeeeee
@Apollorion
@Apollorion Год назад
How stupid is, in order to try factorizing the equation pp-p+1=ppp , subtracting p from both sides of the equation and then: pp-2p+1 = ppp-p = (p-1)^2 = p(pp-1) = (p-1)(p-1) = p(p-1)(p+1) So p=1 (but is that a prime?) or p-1=pp+p The latter solves the same as pp+1=0 so has no real solutions and hence no (extra) primes.
@paperplate9222
@paperplate9222 Год назад
June emoji
@Deejaynerate
@Deejaynerate Год назад
You can't divide 57 by 3, 57 is a prime. Grothendieck told me so.
@adriansison1503
@adriansison1503 Год назад
Basado
@Ali-jb4ns
@Ali-jb4ns Год назад
I have met for the first Bezout Theorem when I was 19 and never really understood it. Thanks to this video and your explanations I finally got it...30 years later. Thank you so much.
@roderickdewar1064
@roderickdewar1064 7 месяцев назад
I boycott all products advertised on RU-vid #BoycottYTads
@damascus21
@damascus21 Год назад
How would you get such a time consuming problem done during a math olympiad??
@MichaelPennMath
@MichaelPennMath Год назад
Michael is really fleshing out the solution, in competition you’d not have to prove the lemma etc. -Stephanie MP Editor
@damascus21
@damascus21 Год назад
​@@MichaelPennMath oh okay! I was assuming the lemma proof was required lol
@shelleyfromyard
@shelleyfromyard Год назад
Going too fast. Got lost really quickly. Math experts make the worst teachers because they assume their audience already understands some fundamentals.
@filipeoliveira7001
@filipeoliveira7001 2 месяца назад
This videos is clearly meant for current math Olympiad competitors, those who aspire to be, or people who are already interested and have knowledge in maths. If you’re clicking on a video with the solution of a national math Olympiad and you have no knowledge on the basics, you’re kinda shooting urself in the foot lol.
@yoav613
@yoav613 Год назад
The new format is great
@emanuellandeholm5657
@emanuellandeholm5657 Год назад
I agree. This is so nice. :)
@someperson188
@someperson188 Год назад
It should be mentioned that n 1 (which is easily checked) in order to conclude (in Case 1) that p | n -1 implies n -1 = px, where x is a natural number.
@AntonioLasoGonzalez
@AntonioLasoGonzalez Год назад
True, but it is an easy check.
@matematicacommarcospaulo
@matematicacommarcospaulo Год назад
Awesome transition at 11:03.... I would like to see it "behind the scenes"
@maldi_tof2910
@maldi_tof2910 Год назад
what an amazing studio!!!
@gtziavelis
@gtziavelis Год назад
343 came up in Domotro's camping/math live-stream last night ~@2:02:40, as a better alternative to 333, because 7*7*7, so that's a cool coincidence.
@whycantiremainanonymous8091
Why such a convoluted proof for the lemma? The prime factorization of ab contains p (because p is a prime), and this guarantees p is a factor of either a, or b, or both.
@YAWTon
@YAWTon Год назад
How would you prove that the prime factorization of ab contains p?
@whycantiremainanonymous8091
@@YAWTonIt's given that p is a prime. It's given that p divides (=is a factor of) ab. If it's prime and it's a factor, it follows that it is a prime factor. Is that proof enough?
@YAWTon
@YAWTon Год назад
@@whycantiremainanonymous8091 No, it isn't, unless if you _assume_ the fundamental theorem of arithmetic, which is a consequence of the lemma that that he proves in the first few minutes of the clip. But of course, in a Math Olympiad Problem you could probably assume knowledge of the fundamental theorem...
@whycantiremainanonymous8091
@@YAWTon Yeah, you should be able to assume it 😃 It's definitely a better known theorem than the one MP used. Otherwise, "Solve without relying on the fundamental theorem of arithmetic" should be typed in boldface all over the problem...
@YAWTon
@YAWTon Год назад
@@whycantiremainanonymous8091 Yes, I agree that one could assume the theorem. But the proof of the lemma wasn't "convoluted".
Далее
just an average recursion...OR IS IT?
18:24
Просмотров 54 тыс.
Как он понял?
00:13
Просмотров 108 тыс.
Watermelon magic box! #shorts by Leisi Crazy
00:20
Просмотров 2,7 млн
Two from Thailand!
26:23
Просмотров 46 тыс.
INTERNATIONAL MATH OLYMPIAD 2023 | Problem 5
15:19
Просмотров 3,7 тыс.
find a closed form for this INTIMIDATING SEQUENCE
22:32
the equation Ramanujan couldn't solve!!
37:03
Просмотров 65 тыс.
what a great "double" functional equation!
13:38
Просмотров 19 тыс.
Twin Prime Conjecture - Numberphile
17:42
Просмотров 790 тыс.
When a mathematician gets bored
10:18
Просмотров 44 тыс.
so you want a VERY HARD math question?!
13:51
Просмотров 1 млн